2
$\begingroup$

X is a non singular projective variety over an infinite field k. How to prove there are no projectives with a surjective map to the structure sheaf O_X in Qch(X) and Coh(X). Coh(X) is the category of coherent sheaves on X.

$\endgroup$
2
  • $\begingroup$ @SashaP Nope. Choices are few since you work in Qch and Coh. $\endgroup$ Dec 31, 2017 at 9:55
  • $\begingroup$ Oh, I see, my bad $\endgroup$
    – SashaP
    Dec 31, 2017 at 10:53

1 Answer 1

11
$\begingroup$

Let $X$ be a projective variety over a field. (A variety is reduced and irreducible.) I'm going to use some elementary properties of the category of quasi-coherent $\mathcal{O}_X$-modules without mention in the discussion below.

If $\dim(X) = 0$, then $X$ is affine and we see that the category of quasi-coherent $\mathcal{O}_X$-modules does have enough projectives. Similarly for the category of coherent $\mathcal{O}_X$-modules.

If $\dim(X) > 0$, then I claim the only projective object of the category of quasi-coherent $\mathcal{O}_X$-modules is the zero object. Namely, let $\mathcal{F}$ be a nonzero quasi-coherent $\mathcal{O}_X$-module. Let $\mathcal{O}_X(1)$ be an ample invertible $\mathcal{O}_X$-module. For some $a \in \mathbf{Z}$ there exists a nonzero map $\mathcal{O}_X(a) \to \mathcal{F}$. Since $X$ is a variety and $\dim(X) > 0$, for every $b < a$ there is no nonzero map $\mathcal{O}_X(a) \to \mathcal{O}_X(b)$. On the other hand, we can find a set $I$, a function $I \to \{b \in \mathbf{Z} \mid b < a\}, i \mapsto b_i$, and a surjection $$ \bigoplus\nolimits_{i \in I} \mathcal{O}_X(b_i) \longrightarrow \mathcal{F} $$ By the remark above this map cannot have a splitting. Thus $\mathcal{F}$ cannot be projective in the category of quasi-coherent $\mathcal{O}_X$-modules. (A fortiori we find that $\mathcal{F}$ is not projective in the category of all $\mathcal{O}_X$-modules.)

For coherent $\mathcal{O}_X$-modules the argument is exactly the same.

$\endgroup$
2
  • 3
    $\begingroup$ +1 for the username. $\endgroup$ Dec 31, 2017 at 15:33
  • 1
    $\begingroup$ Suggestion: accept this answer. $\endgroup$ Jan 1, 2018 at 15:06

Your Answer

By clicking “Post Your Answer”, you agree to our terms of service and acknowledge you have read our privacy policy.

Not the answer you're looking for? Browse other questions tagged or ask your own question.